Field of a uniformly charged disk: integration question( Legendre Generating Function) Off axis Electric Potential from an insulated diskUnderstanding Calculus Notation in PhysicsWhy is this integral for a uniform electric field of a charged plate not evaluating correctly?Calculation of a net electric field for a charged ring - weird integrationElectric field of infinite sheetHow to recover the potential field from Green's function and Poisson's equation for a point charge

Why didn't Thatcher give Hong Kong to Taiwan?

Do we know the problems the University of Manchester's Transistor Computer was intended to solve?

What is the most likely cause of short, quick, and useless reviews?

In chocolate terminology, what is the name of thinly sliced leaf-shaped toppings made from hot, smooth chocolate, used to form flower petals?

Given a specific computer system, is it possible to estimate the actual precise run time of a piece of Assembly code

What is the maximal acceptable delay between pilot's input and flight control surface actuation?

Why do old games use flashing as means of showing damage?

Strange LockTime values in Electrum transactions?

Why don't they build airplanes from 3D printer plastic?

Lumix G7: Raw photos only in 1920x1440, no higher res available

Can my UK debt be collected because I have to return to US?

Does secure hashing imply secure symmetric encryption?

Field of a uniformly charged disk: integration question

Is there anything in the universe that cannot be compressed?

I have two helper functions that are the exact same, one executes and one doesn't. How come?

Meaning of "educating the ice"

Why would a Intel 8080 chip be destroyed if +12 V is connected before -5 V?

My boss says "This will help us better view the utilization of your services." Does this mean my job is ending in this organisation?

Plotting level sets of the form f(x,y,c)==0

Function of the separated, individual solar cells on Telstar 1 and 2? Why were they "special"?

How to check status of Wi-Fi adapter through command line?

Which is the best password hashing algorithm in .NET Core?

Does immunity to non magical damage negate sneak attack damage?

Is mathematics truth?



Field of a uniformly charged disk: integration question


( Legendre Generating Function) Off axis Electric Potential from an insulated diskUnderstanding Calculus Notation in PhysicsWhy is this integral for a uniform electric field of a charged plate not evaluating correctly?Calculation of a net electric field for a charged ring - weird integrationElectric field of infinite sheetHow to recover the potential field from Green's function and Poisson's equation for a point charge






.everyoneloves__top-leaderboard:empty,.everyoneloves__mid-leaderboard:empty,.everyoneloves__bot-mid-leaderboard:empty margin-bottom:0;








2












$begingroup$


In my book (University Physics by Young and Freedman), during solving the common example of finding the electric field along the x-axis from a uniformly charged disk, they arrive at this differential expression:




$$dE_x = 1 over 4piepsilon_02pisigma rx dr over (x^2+r^2)^3/2$$




Then they say:




To find the total field due to all the rings, we integrate $dE_x$ over $r$ from $r=0$ to $r=R$ (not from $-R$ to $R$):



$$E_x = int_0^R 1 over 4piepsilon_0(2pisigma r dr)x over (x^2+r^2)^3/2$$




I am confused as to how they arrived at this line. The right hand side suggests that they integrated both sides of the differential equation with a definite integral, but if that were the case, the left hand side would have been



$$int_0^R dE_x = [E_x]_0^R = R - 0 = R$$



Instead, it looks like they integrated the left hand side of the differential equation with an indefinite integral, so they would get



$$int dE_x = E_x$$



Certainly, integrating one side with an indefinite integral and the other side with a definite integral cannot be a valid step because you are not doing the same thing to both sides.



How can their result be rigorously justified and what is wrong with my reasoning?










share|cite|improve this question











$endgroup$













  • $begingroup$
    You're integrating with dE, not dr, on the left hand side, so the limits of integration can't be the same.
    $endgroup$
    – DanDan0101
    8 hours ago










  • $begingroup$
    Good point. I guess the limits of integration should be the electric field at point $r=0$ and at point $r=R$? $int_E_x(0)^E_x(R) dE_x = E_x(R) - E_x(0)$ This still doesn't make sense to me.
    $endgroup$
    – user109923
    8 hours ago










  • $begingroup$
    But then what does $E_x(r)$ represent? The problem only said that $dE_x$ represents the field component at the point due to a ring.
    $endgroup$
    – user109923
    7 hours ago










  • $begingroup$
    ...and that $vecE=E_xhati$.
    $endgroup$
    – user109923
    7 hours ago


















2












$begingroup$


In my book (University Physics by Young and Freedman), during solving the common example of finding the electric field along the x-axis from a uniformly charged disk, they arrive at this differential expression:




$$dE_x = 1 over 4piepsilon_02pisigma rx dr over (x^2+r^2)^3/2$$




Then they say:




To find the total field due to all the rings, we integrate $dE_x$ over $r$ from $r=0$ to $r=R$ (not from $-R$ to $R$):



$$E_x = int_0^R 1 over 4piepsilon_0(2pisigma r dr)x over (x^2+r^2)^3/2$$




I am confused as to how they arrived at this line. The right hand side suggests that they integrated both sides of the differential equation with a definite integral, but if that were the case, the left hand side would have been



$$int_0^R dE_x = [E_x]_0^R = R - 0 = R$$



Instead, it looks like they integrated the left hand side of the differential equation with an indefinite integral, so they would get



$$int dE_x = E_x$$



Certainly, integrating one side with an indefinite integral and the other side with a definite integral cannot be a valid step because you are not doing the same thing to both sides.



How can their result be rigorously justified and what is wrong with my reasoning?










share|cite|improve this question











$endgroup$













  • $begingroup$
    You're integrating with dE, not dr, on the left hand side, so the limits of integration can't be the same.
    $endgroup$
    – DanDan0101
    8 hours ago










  • $begingroup$
    Good point. I guess the limits of integration should be the electric field at point $r=0$ and at point $r=R$? $int_E_x(0)^E_x(R) dE_x = E_x(R) - E_x(0)$ This still doesn't make sense to me.
    $endgroup$
    – user109923
    8 hours ago










  • $begingroup$
    But then what does $E_x(r)$ represent? The problem only said that $dE_x$ represents the field component at the point due to a ring.
    $endgroup$
    – user109923
    7 hours ago










  • $begingroup$
    ...and that $vecE=E_xhati$.
    $endgroup$
    – user109923
    7 hours ago














2












2








2





$begingroup$


In my book (University Physics by Young and Freedman), during solving the common example of finding the electric field along the x-axis from a uniformly charged disk, they arrive at this differential expression:




$$dE_x = 1 over 4piepsilon_02pisigma rx dr over (x^2+r^2)^3/2$$




Then they say:




To find the total field due to all the rings, we integrate $dE_x$ over $r$ from $r=0$ to $r=R$ (not from $-R$ to $R$):



$$E_x = int_0^R 1 over 4piepsilon_0(2pisigma r dr)x over (x^2+r^2)^3/2$$




I am confused as to how they arrived at this line. The right hand side suggests that they integrated both sides of the differential equation with a definite integral, but if that were the case, the left hand side would have been



$$int_0^R dE_x = [E_x]_0^R = R - 0 = R$$



Instead, it looks like they integrated the left hand side of the differential equation with an indefinite integral, so they would get



$$int dE_x = E_x$$



Certainly, integrating one side with an indefinite integral and the other side with a definite integral cannot be a valid step because you are not doing the same thing to both sides.



How can their result be rigorously justified and what is wrong with my reasoning?










share|cite|improve this question











$endgroup$




In my book (University Physics by Young and Freedman), during solving the common example of finding the electric field along the x-axis from a uniformly charged disk, they arrive at this differential expression:




$$dE_x = 1 over 4piepsilon_02pisigma rx dr over (x^2+r^2)^3/2$$




Then they say:




To find the total field due to all the rings, we integrate $dE_x$ over $r$ from $r=0$ to $r=R$ (not from $-R$ to $R$):



$$E_x = int_0^R 1 over 4piepsilon_0(2pisigma r dr)x over (x^2+r^2)^3/2$$




I am confused as to how they arrived at this line. The right hand side suggests that they integrated both sides of the differential equation with a definite integral, but if that were the case, the left hand side would have been



$$int_0^R dE_x = [E_x]_0^R = R - 0 = R$$



Instead, it looks like they integrated the left hand side of the differential equation with an indefinite integral, so they would get



$$int dE_x = E_x$$



Certainly, integrating one side with an indefinite integral and the other side with a definite integral cannot be a valid step because you are not doing the same thing to both sides.



How can their result be rigorously justified and what is wrong with my reasoning?







electrostatics electric-fields charge integration






share|cite|improve this question















share|cite|improve this question













share|cite|improve this question




share|cite|improve this question








edited 8 hours ago









Aaron Stevens

22.2k4 gold badges41 silver badges78 bronze badges




22.2k4 gold badges41 silver badges78 bronze badges










asked 8 hours ago









user109923user109923

152 bronze badges




152 bronze badges














  • $begingroup$
    You're integrating with dE, not dr, on the left hand side, so the limits of integration can't be the same.
    $endgroup$
    – DanDan0101
    8 hours ago










  • $begingroup$
    Good point. I guess the limits of integration should be the electric field at point $r=0$ and at point $r=R$? $int_E_x(0)^E_x(R) dE_x = E_x(R) - E_x(0)$ This still doesn't make sense to me.
    $endgroup$
    – user109923
    8 hours ago










  • $begingroup$
    But then what does $E_x(r)$ represent? The problem only said that $dE_x$ represents the field component at the point due to a ring.
    $endgroup$
    – user109923
    7 hours ago










  • $begingroup$
    ...and that $vecE=E_xhati$.
    $endgroup$
    – user109923
    7 hours ago

















  • $begingroup$
    You're integrating with dE, not dr, on the left hand side, so the limits of integration can't be the same.
    $endgroup$
    – DanDan0101
    8 hours ago










  • $begingroup$
    Good point. I guess the limits of integration should be the electric field at point $r=0$ and at point $r=R$? $int_E_x(0)^E_x(R) dE_x = E_x(R) - E_x(0)$ This still doesn't make sense to me.
    $endgroup$
    – user109923
    8 hours ago










  • $begingroup$
    But then what does $E_x(r)$ represent? The problem only said that $dE_x$ represents the field component at the point due to a ring.
    $endgroup$
    – user109923
    7 hours ago










  • $begingroup$
    ...and that $vecE=E_xhati$.
    $endgroup$
    – user109923
    7 hours ago
















$begingroup$
You're integrating with dE, not dr, on the left hand side, so the limits of integration can't be the same.
$endgroup$
– DanDan0101
8 hours ago




$begingroup$
You're integrating with dE, not dr, on the left hand side, so the limits of integration can't be the same.
$endgroup$
– DanDan0101
8 hours ago












$begingroup$
Good point. I guess the limits of integration should be the electric field at point $r=0$ and at point $r=R$? $int_E_x(0)^E_x(R) dE_x = E_x(R) - E_x(0)$ This still doesn't make sense to me.
$endgroup$
– user109923
8 hours ago




$begingroup$
Good point. I guess the limits of integration should be the electric field at point $r=0$ and at point $r=R$? $int_E_x(0)^E_x(R) dE_x = E_x(R) - E_x(0)$ This still doesn't make sense to me.
$endgroup$
– user109923
8 hours ago












$begingroup$
But then what does $E_x(r)$ represent? The problem only said that $dE_x$ represents the field component at the point due to a ring.
$endgroup$
– user109923
7 hours ago




$begingroup$
But then what does $E_x(r)$ represent? The problem only said that $dE_x$ represents the field component at the point due to a ring.
$endgroup$
– user109923
7 hours ago












$begingroup$
...and that $vecE=E_xhati$.
$endgroup$
– user109923
7 hours ago





$begingroup$
...and that $vecE=E_xhati$.
$endgroup$
– user109923
7 hours ago











2 Answers
2






active

oldest

votes


















3













$begingroup$

The left hand integral limits should be with respect to the field you get as you add up the contributions to the field due to each charged ring. Therefore, it should start at $0$ and end with the final field. i.e.
$$int_0^E_xtext dE_x' = int_0^R1 over 4piepsilon_02pisigma rx,text dr over (x^2+r^2)^3/2$$



Trivially, the integral on the left side is just the total field $E_x$.




Seeing some of your comments, you seemed to be confused as to what $r$ and $R$ actually represent. It looks like $x$ is the constant distance between the center of the disk and the point at which you are calculating the field at. $r$ is the radius of one of the rings of thickness $text dr$, and $R$ is the radius of the entire disk. Note that you are not determining the field at $r$, since $r$ is not a position in this case.



Your differential relation
$$text dE_x = 1 over 4piepsilon_02pisigma rx,text dr over (x^2+r^2)^3/2=alpha(r),text dr$$
is essentially telling you each ring of radius $r$ and thickness $text dr$ contributes to the field an amount $alpha(r),text dr$. It makes sense that we just need to add up (integrate) all of these values (right integral) to determine the total field (left integral).






share|cite|improve this answer











$endgroup$














  • $begingroup$
    Isn't that a bit like assuming you already know how the answer is going to turn out?
    $endgroup$
    – user109923
    7 hours ago










  • $begingroup$
    As Aaron points out, they are adding contributions to the field from thin rings of charge located in the disk, each of radius r and thickness dr. Each little segment of a ring contributes at an angle from the x axis. For the x component you multiply by x/(x^2 + r^2)^(1/2). The charge in each ring depends on the charge density and area: σ(2 π r dr). The field drops off with the distance squared: (x^2 + r^2). The limits reflect the distribution of charged rings.
    $endgroup$
    – R.W. Bird
    7 hours ago










  • $begingroup$
    @user109923 not at all. Just because I know that adding up all of the field contributions gives me the total field it doesn't mean I know what that field is. That is what the right side integral is for.
    $endgroup$
    – Aaron Stevens
    7 hours ago










  • $begingroup$
    @user109923 Is there anything that is unclear to you?
    $endgroup$
    – Aaron Stevens
    4 hours ago


















0













$begingroup$


But then what does $E_x(r)$ represent? The problem only said that $dE_x$
represents the field component at the point due to a ring




$E_x(r)$ is a poor notation here since it certainly looks like the $x$ component of the electric field as a function of the radial coordinate $r$. Further, it is clear from the integral that $E_x = E_x(x)$.



Here, $dE_x$ or $dE_x|_r$ is the contribution to the total electric field (at coordinate $x$ on the $x$-axis) due an infinitesimal ring of charge between $r$ and $r + dr$.



If it helps, consider the approximation by finite rings of charge located between $r$ and $r + Delta r$. Let there be $N$ such rings such that



$$Delta r = fracRN$$



and



$$r_n = (n-1)Delta r$$



To find the total electric field due to the contributions $E_x,n$ from the $N$ rings making up the disk, we simply sum the contributions (linearity of electric field).



$$E_x = sum_n=1^NE_x,n$$



In the limit $Nrightarrowinfty$, the contributions become infinitesimal $E_x,nrightarrow dE_x|_r$, and the sum becomes an integral.






share|cite|improve this answer











$endgroup$

















    Your Answer








    StackExchange.ready(function()
    var channelOptions =
    tags: "".split(" "),
    id: "151"
    ;
    initTagRenderer("".split(" "), "".split(" "), channelOptions);

    StackExchange.using("externalEditor", function()
    // Have to fire editor after snippets, if snippets enabled
    if (StackExchange.settings.snippets.snippetsEnabled)
    StackExchange.using("snippets", function()
    createEditor();
    );

    else
    createEditor();

    );

    function createEditor()
    StackExchange.prepareEditor(
    heartbeatType: 'answer',
    autoActivateHeartbeat: false,
    convertImagesToLinks: false,
    noModals: true,
    showLowRepImageUploadWarning: true,
    reputationToPostImages: null,
    bindNavPrevention: true,
    postfix: "",
    imageUploader:
    brandingHtml: "Powered by u003ca class="icon-imgur-white" href="https://imgur.com/"u003eu003c/au003e",
    contentPolicyHtml: "User contributions licensed under u003ca href="https://creativecommons.org/licenses/by-sa/3.0/"u003ecc by-sa 3.0 with attribution requiredu003c/au003e u003ca href="https://stackoverflow.com/legal/content-policy"u003e(content policy)u003c/au003e",
    allowUrls: true
    ,
    noCode: true, onDemand: true,
    discardSelector: ".discard-answer"
    ,immediatelyShowMarkdownHelp:true
    );



    );













    draft saved

    draft discarded


















    StackExchange.ready(
    function ()
    StackExchange.openid.initPostLogin('.new-post-login', 'https%3a%2f%2fphysics.stackexchange.com%2fquestions%2f499914%2ffield-of-a-uniformly-charged-disk-integration-question%23new-answer', 'question_page');

    );

    Post as a guest















    Required, but never shown

























    2 Answers
    2






    active

    oldest

    votes








    2 Answers
    2






    active

    oldest

    votes









    active

    oldest

    votes






    active

    oldest

    votes









    3













    $begingroup$

    The left hand integral limits should be with respect to the field you get as you add up the contributions to the field due to each charged ring. Therefore, it should start at $0$ and end with the final field. i.e.
    $$int_0^E_xtext dE_x' = int_0^R1 over 4piepsilon_02pisigma rx,text dr over (x^2+r^2)^3/2$$



    Trivially, the integral on the left side is just the total field $E_x$.




    Seeing some of your comments, you seemed to be confused as to what $r$ and $R$ actually represent. It looks like $x$ is the constant distance between the center of the disk and the point at which you are calculating the field at. $r$ is the radius of one of the rings of thickness $text dr$, and $R$ is the radius of the entire disk. Note that you are not determining the field at $r$, since $r$ is not a position in this case.



    Your differential relation
    $$text dE_x = 1 over 4piepsilon_02pisigma rx,text dr over (x^2+r^2)^3/2=alpha(r),text dr$$
    is essentially telling you each ring of radius $r$ and thickness $text dr$ contributes to the field an amount $alpha(r),text dr$. It makes sense that we just need to add up (integrate) all of these values (right integral) to determine the total field (left integral).






    share|cite|improve this answer











    $endgroup$














    • $begingroup$
      Isn't that a bit like assuming you already know how the answer is going to turn out?
      $endgroup$
      – user109923
      7 hours ago










    • $begingroup$
      As Aaron points out, they are adding contributions to the field from thin rings of charge located in the disk, each of radius r and thickness dr. Each little segment of a ring contributes at an angle from the x axis. For the x component you multiply by x/(x^2 + r^2)^(1/2). The charge in each ring depends on the charge density and area: σ(2 π r dr). The field drops off with the distance squared: (x^2 + r^2). The limits reflect the distribution of charged rings.
      $endgroup$
      – R.W. Bird
      7 hours ago










    • $begingroup$
      @user109923 not at all. Just because I know that adding up all of the field contributions gives me the total field it doesn't mean I know what that field is. That is what the right side integral is for.
      $endgroup$
      – Aaron Stevens
      7 hours ago










    • $begingroup$
      @user109923 Is there anything that is unclear to you?
      $endgroup$
      – Aaron Stevens
      4 hours ago















    3













    $begingroup$

    The left hand integral limits should be with respect to the field you get as you add up the contributions to the field due to each charged ring. Therefore, it should start at $0$ and end with the final field. i.e.
    $$int_0^E_xtext dE_x' = int_0^R1 over 4piepsilon_02pisigma rx,text dr over (x^2+r^2)^3/2$$



    Trivially, the integral on the left side is just the total field $E_x$.




    Seeing some of your comments, you seemed to be confused as to what $r$ and $R$ actually represent. It looks like $x$ is the constant distance between the center of the disk and the point at which you are calculating the field at. $r$ is the radius of one of the rings of thickness $text dr$, and $R$ is the radius of the entire disk. Note that you are not determining the field at $r$, since $r$ is not a position in this case.



    Your differential relation
    $$text dE_x = 1 over 4piepsilon_02pisigma rx,text dr over (x^2+r^2)^3/2=alpha(r),text dr$$
    is essentially telling you each ring of radius $r$ and thickness $text dr$ contributes to the field an amount $alpha(r),text dr$. It makes sense that we just need to add up (integrate) all of these values (right integral) to determine the total field (left integral).






    share|cite|improve this answer











    $endgroup$














    • $begingroup$
      Isn't that a bit like assuming you already know how the answer is going to turn out?
      $endgroup$
      – user109923
      7 hours ago










    • $begingroup$
      As Aaron points out, they are adding contributions to the field from thin rings of charge located in the disk, each of radius r and thickness dr. Each little segment of a ring contributes at an angle from the x axis. For the x component you multiply by x/(x^2 + r^2)^(1/2). The charge in each ring depends on the charge density and area: σ(2 π r dr). The field drops off with the distance squared: (x^2 + r^2). The limits reflect the distribution of charged rings.
      $endgroup$
      – R.W. Bird
      7 hours ago










    • $begingroup$
      @user109923 not at all. Just because I know that adding up all of the field contributions gives me the total field it doesn't mean I know what that field is. That is what the right side integral is for.
      $endgroup$
      – Aaron Stevens
      7 hours ago










    • $begingroup$
      @user109923 Is there anything that is unclear to you?
      $endgroup$
      – Aaron Stevens
      4 hours ago













    3














    3










    3







    $begingroup$

    The left hand integral limits should be with respect to the field you get as you add up the contributions to the field due to each charged ring. Therefore, it should start at $0$ and end with the final field. i.e.
    $$int_0^E_xtext dE_x' = int_0^R1 over 4piepsilon_02pisigma rx,text dr over (x^2+r^2)^3/2$$



    Trivially, the integral on the left side is just the total field $E_x$.




    Seeing some of your comments, you seemed to be confused as to what $r$ and $R$ actually represent. It looks like $x$ is the constant distance between the center of the disk and the point at which you are calculating the field at. $r$ is the radius of one of the rings of thickness $text dr$, and $R$ is the radius of the entire disk. Note that you are not determining the field at $r$, since $r$ is not a position in this case.



    Your differential relation
    $$text dE_x = 1 over 4piepsilon_02pisigma rx,text dr over (x^2+r^2)^3/2=alpha(r),text dr$$
    is essentially telling you each ring of radius $r$ and thickness $text dr$ contributes to the field an amount $alpha(r),text dr$. It makes sense that we just need to add up (integrate) all of these values (right integral) to determine the total field (left integral).






    share|cite|improve this answer











    $endgroup$



    The left hand integral limits should be with respect to the field you get as you add up the contributions to the field due to each charged ring. Therefore, it should start at $0$ and end with the final field. i.e.
    $$int_0^E_xtext dE_x' = int_0^R1 over 4piepsilon_02pisigma rx,text dr over (x^2+r^2)^3/2$$



    Trivially, the integral on the left side is just the total field $E_x$.




    Seeing some of your comments, you seemed to be confused as to what $r$ and $R$ actually represent. It looks like $x$ is the constant distance between the center of the disk and the point at which you are calculating the field at. $r$ is the radius of one of the rings of thickness $text dr$, and $R$ is the radius of the entire disk. Note that you are not determining the field at $r$, since $r$ is not a position in this case.



    Your differential relation
    $$text dE_x = 1 over 4piepsilon_02pisigma rx,text dr over (x^2+r^2)^3/2=alpha(r),text dr$$
    is essentially telling you each ring of radius $r$ and thickness $text dr$ contributes to the field an amount $alpha(r),text dr$. It makes sense that we just need to add up (integrate) all of these values (right integral) to determine the total field (left integral).







    share|cite|improve this answer














    share|cite|improve this answer



    share|cite|improve this answer








    edited 4 hours ago

























    answered 8 hours ago









    Aaron StevensAaron Stevens

    22.2k4 gold badges41 silver badges78 bronze badges




    22.2k4 gold badges41 silver badges78 bronze badges














    • $begingroup$
      Isn't that a bit like assuming you already know how the answer is going to turn out?
      $endgroup$
      – user109923
      7 hours ago










    • $begingroup$
      As Aaron points out, they are adding contributions to the field from thin rings of charge located in the disk, each of radius r and thickness dr. Each little segment of a ring contributes at an angle from the x axis. For the x component you multiply by x/(x^2 + r^2)^(1/2). The charge in each ring depends on the charge density and area: σ(2 π r dr). The field drops off with the distance squared: (x^2 + r^2). The limits reflect the distribution of charged rings.
      $endgroup$
      – R.W. Bird
      7 hours ago










    • $begingroup$
      @user109923 not at all. Just because I know that adding up all of the field contributions gives me the total field it doesn't mean I know what that field is. That is what the right side integral is for.
      $endgroup$
      – Aaron Stevens
      7 hours ago










    • $begingroup$
      @user109923 Is there anything that is unclear to you?
      $endgroup$
      – Aaron Stevens
      4 hours ago
















    • $begingroup$
      Isn't that a bit like assuming you already know how the answer is going to turn out?
      $endgroup$
      – user109923
      7 hours ago










    • $begingroup$
      As Aaron points out, they are adding contributions to the field from thin rings of charge located in the disk, each of radius r and thickness dr. Each little segment of a ring contributes at an angle from the x axis. For the x component you multiply by x/(x^2 + r^2)^(1/2). The charge in each ring depends on the charge density and area: σ(2 π r dr). The field drops off with the distance squared: (x^2 + r^2). The limits reflect the distribution of charged rings.
      $endgroup$
      – R.W. Bird
      7 hours ago










    • $begingroup$
      @user109923 not at all. Just because I know that adding up all of the field contributions gives me the total field it doesn't mean I know what that field is. That is what the right side integral is for.
      $endgroup$
      – Aaron Stevens
      7 hours ago










    • $begingroup$
      @user109923 Is there anything that is unclear to you?
      $endgroup$
      – Aaron Stevens
      4 hours ago















    $begingroup$
    Isn't that a bit like assuming you already know how the answer is going to turn out?
    $endgroup$
    – user109923
    7 hours ago




    $begingroup$
    Isn't that a bit like assuming you already know how the answer is going to turn out?
    $endgroup$
    – user109923
    7 hours ago












    $begingroup$
    As Aaron points out, they are adding contributions to the field from thin rings of charge located in the disk, each of radius r and thickness dr. Each little segment of a ring contributes at an angle from the x axis. For the x component you multiply by x/(x^2 + r^2)^(1/2). The charge in each ring depends on the charge density and area: σ(2 π r dr). The field drops off with the distance squared: (x^2 + r^2). The limits reflect the distribution of charged rings.
    $endgroup$
    – R.W. Bird
    7 hours ago




    $begingroup$
    As Aaron points out, they are adding contributions to the field from thin rings of charge located in the disk, each of radius r and thickness dr. Each little segment of a ring contributes at an angle from the x axis. For the x component you multiply by x/(x^2 + r^2)^(1/2). The charge in each ring depends on the charge density and area: σ(2 π r dr). The field drops off with the distance squared: (x^2 + r^2). The limits reflect the distribution of charged rings.
    $endgroup$
    – R.W. Bird
    7 hours ago












    $begingroup$
    @user109923 not at all. Just because I know that adding up all of the field contributions gives me the total field it doesn't mean I know what that field is. That is what the right side integral is for.
    $endgroup$
    – Aaron Stevens
    7 hours ago




    $begingroup$
    @user109923 not at all. Just because I know that adding up all of the field contributions gives me the total field it doesn't mean I know what that field is. That is what the right side integral is for.
    $endgroup$
    – Aaron Stevens
    7 hours ago












    $begingroup$
    @user109923 Is there anything that is unclear to you?
    $endgroup$
    – Aaron Stevens
    4 hours ago




    $begingroup$
    @user109923 Is there anything that is unclear to you?
    $endgroup$
    – Aaron Stevens
    4 hours ago













    0













    $begingroup$


    But then what does $E_x(r)$ represent? The problem only said that $dE_x$
    represents the field component at the point due to a ring




    $E_x(r)$ is a poor notation here since it certainly looks like the $x$ component of the electric field as a function of the radial coordinate $r$. Further, it is clear from the integral that $E_x = E_x(x)$.



    Here, $dE_x$ or $dE_x|_r$ is the contribution to the total electric field (at coordinate $x$ on the $x$-axis) due an infinitesimal ring of charge between $r$ and $r + dr$.



    If it helps, consider the approximation by finite rings of charge located between $r$ and $r + Delta r$. Let there be $N$ such rings such that



    $$Delta r = fracRN$$



    and



    $$r_n = (n-1)Delta r$$



    To find the total electric field due to the contributions $E_x,n$ from the $N$ rings making up the disk, we simply sum the contributions (linearity of electric field).



    $$E_x = sum_n=1^NE_x,n$$



    In the limit $Nrightarrowinfty$, the contributions become infinitesimal $E_x,nrightarrow dE_x|_r$, and the sum becomes an integral.






    share|cite|improve this answer











    $endgroup$



















      0













      $begingroup$


      But then what does $E_x(r)$ represent? The problem only said that $dE_x$
      represents the field component at the point due to a ring




      $E_x(r)$ is a poor notation here since it certainly looks like the $x$ component of the electric field as a function of the radial coordinate $r$. Further, it is clear from the integral that $E_x = E_x(x)$.



      Here, $dE_x$ or $dE_x|_r$ is the contribution to the total electric field (at coordinate $x$ on the $x$-axis) due an infinitesimal ring of charge between $r$ and $r + dr$.



      If it helps, consider the approximation by finite rings of charge located between $r$ and $r + Delta r$. Let there be $N$ such rings such that



      $$Delta r = fracRN$$



      and



      $$r_n = (n-1)Delta r$$



      To find the total electric field due to the contributions $E_x,n$ from the $N$ rings making up the disk, we simply sum the contributions (linearity of electric field).



      $$E_x = sum_n=1^NE_x,n$$



      In the limit $Nrightarrowinfty$, the contributions become infinitesimal $E_x,nrightarrow dE_x|_r$, and the sum becomes an integral.






      share|cite|improve this answer











      $endgroup$

















        0














        0










        0







        $begingroup$


        But then what does $E_x(r)$ represent? The problem only said that $dE_x$
        represents the field component at the point due to a ring




        $E_x(r)$ is a poor notation here since it certainly looks like the $x$ component of the electric field as a function of the radial coordinate $r$. Further, it is clear from the integral that $E_x = E_x(x)$.



        Here, $dE_x$ or $dE_x|_r$ is the contribution to the total electric field (at coordinate $x$ on the $x$-axis) due an infinitesimal ring of charge between $r$ and $r + dr$.



        If it helps, consider the approximation by finite rings of charge located between $r$ and $r + Delta r$. Let there be $N$ such rings such that



        $$Delta r = fracRN$$



        and



        $$r_n = (n-1)Delta r$$



        To find the total electric field due to the contributions $E_x,n$ from the $N$ rings making up the disk, we simply sum the contributions (linearity of electric field).



        $$E_x = sum_n=1^NE_x,n$$



        In the limit $Nrightarrowinfty$, the contributions become infinitesimal $E_x,nrightarrow dE_x|_r$, and the sum becomes an integral.






        share|cite|improve this answer











        $endgroup$




        But then what does $E_x(r)$ represent? The problem only said that $dE_x$
        represents the field component at the point due to a ring




        $E_x(r)$ is a poor notation here since it certainly looks like the $x$ component of the electric field as a function of the radial coordinate $r$. Further, it is clear from the integral that $E_x = E_x(x)$.



        Here, $dE_x$ or $dE_x|_r$ is the contribution to the total electric field (at coordinate $x$ on the $x$-axis) due an infinitesimal ring of charge between $r$ and $r + dr$.



        If it helps, consider the approximation by finite rings of charge located between $r$ and $r + Delta r$. Let there be $N$ such rings such that



        $$Delta r = fracRN$$



        and



        $$r_n = (n-1)Delta r$$



        To find the total electric field due to the contributions $E_x,n$ from the $N$ rings making up the disk, we simply sum the contributions (linearity of electric field).



        $$E_x = sum_n=1^NE_x,n$$



        In the limit $Nrightarrowinfty$, the contributions become infinitesimal $E_x,nrightarrow dE_x|_r$, and the sum becomes an integral.







        share|cite|improve this answer














        share|cite|improve this answer



        share|cite|improve this answer








        edited 4 hours ago

























        answered 4 hours ago









        Alfred CentauriAlfred Centauri

        51.1k3 gold badges54 silver badges166 bronze badges




        51.1k3 gold badges54 silver badges166 bronze badges






























            draft saved

            draft discarded
















































            Thanks for contributing an answer to Physics Stack Exchange!


            • Please be sure to answer the question. Provide details and share your research!

            But avoid


            • Asking for help, clarification, or responding to other answers.

            • Making statements based on opinion; back them up with references or personal experience.

            Use MathJax to format equations. MathJax reference.


            To learn more, see our tips on writing great answers.




            draft saved


            draft discarded














            StackExchange.ready(
            function ()
            StackExchange.openid.initPostLogin('.new-post-login', 'https%3a%2f%2fphysics.stackexchange.com%2fquestions%2f499914%2ffield-of-a-uniformly-charged-disk-integration-question%23new-answer', 'question_page');

            );

            Post as a guest















            Required, but never shown





















































            Required, but never shown














            Required, but never shown












            Required, but never shown







            Required, but never shown

































            Required, but never shown














            Required, but never shown












            Required, but never shown







            Required, but never shown







            Popular posts from this blog

            ParseJSON using SSJSUsing AMPscript with SSJS ActivitiesHow to resubscribe a user in Marketing cloud using SSJS?Pulling Subscriber Status from Lists using SSJSRetrieving Emails using SSJSProblem in updating DE using SSJSUsing SSJS to send single email in Marketing CloudError adding EmailSendDefinition using SSJS

            Кампала Садржај Географија Географија Историја Становништво Привреда Партнерски градови Референце Спољашње везе Мени за навигацију0°11′ СГШ; 32°20′ ИГД / 0.18° СГШ; 32.34° ИГД / 0.18; 32.340°11′ СГШ; 32°20′ ИГД / 0.18° СГШ; 32.34° ИГД / 0.18; 32.34МедијиПодациЗванични веб-сајту

            19. јануар Садржај Догађаји Рођења Смрти Празници и дани сећања Види још Референце Мени за навигацијуу